Find Extrema of f(x) = x^3+y^3+3x^2-18y^2+81y+5

  • Thread starter Thread starter kliker
  • Start date Start date
  • Tags Tags
    Extrema Function
Click For Summary
To find the extrema of the function f(x) = x^3 + y^3 + 3x^2 - 18y^2 + 81y + 5, the gradient must equal zero, leading to the equations 3x^2 + 6x = 0 and 3y^2 - 36y + 81 = 0. The solutions for x are x = 0 or x = -2, while the correct y values need verification, as y = 2 does not satisfy the equation. It is essential to check all potential critical points by substituting back into the function or using the second derivative test. Ultimately, any value of x can be paired with any value of y, resulting in four critical points to evaluate for extrema.
kliker
Messages
102
Reaction score
0
given this function

f(x) = x^3+y^3+3x^2-18y^2+81y+5

i should i find the extrema(i hope this is how it is called in english)

so we should have gradf = 0

hence
3x^2+6x = 0 and 3y^2-36y+81 = 0

here i get x = 0 or x = -2 and y = 54/9 or y = 2

now what i want to ask is, which points will i have to take in order to check of extrema?

it will be
P1 = (0,54/9) and P2 = (-2,2)?
P1 = (0,2) and P2 = (-2,54/9)?

which one is the correct?

Thanks in advance
 
Physics news on Phys.org
i don't think your y values are correct... try subtituting them back in
3y^2-36y+81 = 3(y^2 -12 +27) = 0

you will need to check all of the points, either by subtituting in or a 2nd derivative check
 
First, as lanedance said, your solution for y is incorrect (it is easy to see that y= 2 does not satify the equation: 3(4)- 36(2)+ 81= -72+ 93= 21, not 0).

Second, once you have correct solutions for x and y, since the two equations are completely separate, any value of x can be used with any value of y- there are four critical points.
 
thanks a lot for your help :)
 
Question: A clock's minute hand has length 4 and its hour hand has length 3. What is the distance between the tips at the moment when it is increasing most rapidly?(Putnam Exam Question) Answer: Making assumption that both the hands moves at constant angular velocities, the answer is ## \sqrt{7} .## But don't you think this assumption is somewhat doubtful and wrong?

Similar threads

Replies
5
Views
1K
Replies
2
Views
2K
  • · Replies 7 ·
Replies
7
Views
2K
  • · Replies 11 ·
Replies
11
Views
2K
Replies
11
Views
2K
  • · Replies 7 ·
Replies
7
Views
3K
  • · Replies 6 ·
Replies
6
Views
2K
Replies
10
Views
1K
  • · Replies 11 ·
Replies
11
Views
2K
  • · Replies 11 ·
Replies
11
Views
2K